\documentclass[11pt]{article} \usepackage{amsthm,amsmath,amsfonts,amssymb} \usepackage[ruled,noline,noend]{algorithm2e} \usepackage[margin=1in]{geometry} \usepackage{enumerate} %%% Update the following commands with your name, ID, and acknowledgments. \newcommand{\YourName}{REPLACE WITH YOUR NAME} \newcommand{\YourStudentID}{REPLACE WITH ID NUMBER} \newcommand{\YourAck}{REPLACE this text with a full acknowledgement of all sources (people you discussed the question with and/or online/text sources you consulted) used while completing this question. If you completed the question without consulting any sources, say so here explicitly.} \newenvironment{solution}{\paragraph{Solution.}}{} \title{ \vspace{-0.8in} \normalsize \begin{flushright} \YourName \\ \YourStudentID \end{flushright} \rule{\textwidth}{.5pt}\\[0.4cm] \huge Assignment 1: Question 2 \\ \rule{\textwidth}{2pt} \vspace{-1in} } \date{} \begin{document} \maketitle \paragraph{Acknowledgments.} \YourAck %%%%%%%%%%%%%%%%%%%%%%%%%%%%%%%%%%%% \section*{Solving recurrences} Solve the following recurrence relations to obtain a closed-form big-$\Theta$ expression for $T(n)$. In each question, assume $T(c)$ is bounded by a constant for any small constant $c$. \begin{enumerate}[(a)] \item $T(n) = 9\,T(\frac{n}{3}) + n^2$ \begin{solution} (ENTER YOUR SOLUTION HERE.) \end{solution} \item $T(n) = 4\,T(\frac{n}{4}) + n \log n$ \begin{solution} (ENTER YOUR SOLUTION HERE.) \end{solution} \item $T(n) = T(\frac{n}{4}) + T(\frac{3n}{4}) + n$ \begin{solution} (ENTER YOUR SOLUTION HERE.) \end{solution} \item $T(n) = \sqrt{n} \cdot T(\sqrt{n}) + n$ \\ \emph{Hint.} The correct expression is somewhere between $\Omega(n)$ and $O(n \log n)$. \begin{solution} (ENTER YOUR SOLUTION HERE.) \end{solution} \end{enumerate} \end{document}